Double Integrals: Finding the Volume of a Solid Using Polar Coordinates

  • Thread starter Thread starter Lamebert
  • Start date Start date
  • Tags Tags
    Integrals Polar
Click For Summary
The discussion focuses on finding the volume of a solid enclosed by the plane z = 2 and the paraboloid z = 8 - 6x² - 6y² using polar coordinates. The intersection of the two surfaces is determined to be a circle of radius 1 on the plane z = 2. The appropriate domain for the double integral is established as 0 ≤ r ≤ 1 and 0 ≤ θ ≤ 2π. There is a caution about potentially including the volume below z = 2 within the circle, which could lead to inaccuracies in the final volume calculation. The user is encouraged to verify their integration setup to avoid this error.
Lamebert
Messages
39
Reaction score
1

Homework Statement



The plane z = 2 and the paraboloid z = 8 − 6x2 − 6y2 enclose a solid. Use polar coordinates to find the volume of this solid.

Homework Equations



∫∫R f(x,y) dA = ∫βαba f(rcosθ, rsinθ) r dr dθ

The Attempt at a Solution



z = 2, z = 8 − 6x2 − 6y2

Setting these two equal, we can find where the two functions intersect.

2 = 8 − 6x2 − 6y2

so

0 = 6 − 6x2 − 6y2

Solving for x and y, we get

1 = x2 + y2

So the intersection is a circle of radius 1 on the plane z = 2.

Knowing this, we can write the domain of x and y both in terms of r and θ:

{ r,θ | 0 ≤ r ≤ 1, 0 ≤ θ ≤ 2∏}

Using this domain, I set up my double integral with the above layout found in my provided equations section. I don't feel like typing all the integrations out, but is my above process wrong? If not, I can focus on finding errors in my integration and ask further questions as needed.

Thanks.
 
Last edited:
Physics news on Phys.org
Looks to me that you are in danger of including the volume below z=2 (within the circle).
 
Question: A clock's minute hand has length 4 and its hour hand has length 3. What is the distance between the tips at the moment when it is increasing most rapidly?(Putnam Exam Question) Answer: Making assumption that both the hands moves at constant angular velocities, the answer is ## \sqrt{7} .## But don't you think this assumption is somewhat doubtful and wrong?

Similar threads

  • · Replies 19 ·
Replies
19
Views
2K
  • · Replies 9 ·
Replies
9
Views
2K
  • · Replies 2 ·
Replies
2
Views
2K
Replies
6
Views
2K
  • · Replies 3 ·
Replies
3
Views
3K
Replies
3
Views
1K
Replies
4
Views
1K
  • · Replies 2 ·
Replies
2
Views
1K
  • · Replies 3 ·
Replies
3
Views
1K
  • · Replies 7 ·
Replies
7
Views
2K